For the velocity distribution of Prob. 4.10, (a)(a) check continuity. (b)(b) Are the Navier-Stokes equations valid? (c)(c) If so, find p(x,y)p(x,y) if the pressure at the origin is p0p0​.

Answers

Answer 1

There is continuity in the flow as the velocity distribution is constant in the x-direction, but it can vary in the y-direction.The Navier-Stokes equations are thus valid. Finally, p(x,y)p(x,y) if the pressure at the origin is p0p0​ is $$ p(x,y) = p_0 - \rho V^2 \frac{xy}{h^2} $$.

(a) Check continuity:Let v (x) be the velocity distribution. By the equation of continuity, we have the following:$$ \frac{d}{dx}v(x) =0 $$Thus, the velocity distribution is constant in the x-direction, but it can vary in the y-direction. Therefore, there is continuity in the flow.

(b)The Navier-Stokes equations are valid. (c) write down the Navier-Stokes equations:

$$ \frac{\partial v}{\partial t} + v \frac{\partial v}{\partial x} + w \frac{\partial v}{\partial y} = - \frac{1}{\rho}\frac{\partial p}{\partial x} + g_x $$$$ \frac{\partial w}{\partial t} + v \frac{\partial w}{\partial x} + w \frac{\partial w}{\partial y} = - \frac{1}{\rho}\frac{\partial p}{\partial y} + g_y $$.Since there is no gravity, we have that g x = g y = 0. Then, we can rewrite the equations as follows:

$$ \frac{\partial v}{\partial t} + v \frac{\partial v}{\partial x} + w \frac{\partial v}{\partial y} = - \frac{1}{\rho}\frac{\partial p}{\partial x} $$$$ \frac{\partial w}{\partial t} + v \frac{\partial w}{\partial x} + w \frac{\partial w}{\partial y} = - \frac{1}{\rho}\frac{\partial p}{\partial y} $$We also have the following:$$ \frac{\partial v}{\partial x} + \frac{\partial w}{\partial y} = 0 $$.From the velocity distribution, we have that:$$ v(x,y) = - \frac{V}{h}y $$where V is the maximum velocity at the centerline, and h is the half-width of the channel.

Then, we can write the pressure distribution as follows:$$ \frac{\partial p}{\partial x} = - \rho V^2 \frac{y}{h^2} $$Integrating with respect to x, we obtain:$$ p(x,y) = p_0 - \rho V^2 \frac{xy}{h^2} $$where p 0 is the pressure at the origin.

More on velocity distribution: https://brainly.com/question/29991330

#SPJ11


Related Questions

Part GNow, using the results of Part F, find the total kinetic energy K of the system. Remember that both particles rotate about the y axis.
Express your answer in terms of m, ω, and r.

Answers

Answer:

K = (11*m*(ω*r)^2)/2

Explanation:

Not surprisingly, the formulas K = (1/2)*I*ω^2 and K = (1/2)*m*v^2 give the same result. They should, of course, since the rotational kinetic energy of a system of particles is simply the sum of the kinetic energies of the individual particles making up the system.

A child sits on the floor and twirls a 0.100 kg ball connected to a 1.0 m string in a horizontal cirlce along the floor with a speed of 2.0 m/s. What forces are acting on the floor? All of the above A. Only gravity force and tension B. Tension C. Force due to gravity weight D. normal force

Answers

The forces acting on the floor when a child twirls a 0.100 kg ball connected to a 1.0 m string in a horizontal circle along the floor with a speed of 2.0 m/s are: A. Only gravity force and tension.

These forces include the force due to gravity (weight) acting vertically downwards and the tension in the string acting horizontally.

The formula for centripetal acceleration is: a = v²/r

where - a is the centripetal acceleration

            - v is the velocity

            - r is the radius of the circle

The force of tension is the force that arises when an object is pulled or stretched by a rope or cable that is connected to it. In this case, the ball is connected to the string and the string is being pulled to maintain the circular motion. Therefore, the force of tension is present.

The force of gravity weight is the force exerted by the Earth's gravitational field on an object. It is calculated using the formula: W = mg

where - W is the weight of the object

            - m is the mass of the object  

            - g is the acceleration due to gravity (9.8 m/s²)Therefore, the force of gravity weight is also present in this scenario.
The normal force acts on the ball to balance the force due to gravity, but it acts perpendicular to the floor and does not directly affect it.

Learn more about tension here:

https://brainly.com/question/24994188

#SPJ11

Which of the following values are needed to solve for the heat capacity of a heated unknown solid when it is placed in a coffee cup calorimeter? Select all that apply.
A. The initial temperature of the water
B. The initial temperature of the solid
C. The mass of the solid
D. The mass of the water in the cup
E. The final temperature of the system

Answers

The initial temperature of the solid, the mass of the solid, the mass of the water in the cup, and the final temperature of the system are needed to solve for the heat capacity of a heated unknown solid when it is placed in a coffee cup calorimeter.

The heat capacity is the amount of heat required to raise the temperature of a system by one degree Celsius (or one Kelvin). When an unknown solid is heated and placed in a coffee cup calorimeter, the following values are needed to calculate its heat capacity: Initial temperature of the solid, Mass of the solid, Mass of the water in the cup, Final temperature of the system.

The calorimeter is used to measure the amount of heat gained or lost by the system (the solid and the water in the cup) during the process. The change in temperature of the system is used to calculate the heat gained or lost by the solid.

Therefore, the initial temperature of the solid, the mass of the solid, the mass of the water in the cup, and the final temperature of the system are needed to solve for the heat capacity of the heated unknown solid.

Learn more about specific heat capacity here:

https://brainly.com/question/27991746

#SPJ11

explain how angela can calculate the frictional torque applied to the wheel. state what apparatus is to be used.

Answers

Angela can calculate the frictional torque applied to the wheel by using a torque wrench. The torque wrench measures the amount of torque needed to rotate the wheel, which is equal to the frictional torque.

Calculating the frictional torque applied to a wheel.

To calculate the frictional torque applied to a wheel, Angela can use a torque wrench. A torque wrench is a tool that is used to measure the amount of torque required to rotate the wheel.

When the wheel is turned with the torque wrench, the amount of torque needed to rotate it is equal to the frictional torque that the wheel is experiencing.

By measuring the torque required to rotate the wheel, Angela can calculate the frictional torque accurately.

Learn more about torque here:

https://brainly.com/question/20691242

#SPJ1

as you found in part a, your weight will be greater than normal when the elevator is moving upward with increasing speed. for what other motion would your weight also be greater than your normal weight?

Answers

Answer:

1)  F = M a      accelerating forward would cause a seeming increase weight.

2) F = m v^2 / R    a person moving in a circular arc would have an apparent component of  weight towards the center of rotation (in addition to his normal vertical component of weight).

a truck collides with a car, and during the collision, the net force on each vehicle is essentially the force exerted by the other. suppose the mass of the car is 550 kg, the mass of the truck is 2200 kg, and the magnitude of the truck's acceleration is 10 m / s 2 10m/s 2 . find the magnitude of the car's acceleration.

Answers

Answer: The magnitude of the car's acceleration is 40m/s²

Explanation:

According to Newton's third law of motion, for every action, there is an equal and opposite reaction. Therefore, the force exerted by the car on the truck is similar to the force exerted by the truck on the car in magnitude and opposite direction.

To begin, we must calculate the force exerted by the truck on the car. The force exerted by the truck on the car is given as:

F = maF = (mass of car) × (acceleration of car)

Now, we need to calculate the acceleration of the car.

a = F/m a = [(mass of truck) × (acceleration of truck)] / (mass of car)

Substituting the given values, we get:

a = [(2200 kg) × (10 m/s²)] / (550 kg) = 40 m/s²

Therefore, the magnitude of the car's acceleration is 40 m/s².

Learn more about acceleration here:

https://brainly.com/question/460763

#SPJ11

of the three states of matter, which one has the most kinetic energy?

Answers

Of the three states of matter (solid, liquid, and gas), gas has the most kinetic energy. This is because the particles in a gas have the highest average speed compared to the particles in solids and liquids.

In a gas, the particles are in constant motion, colliding with each other and the walls of the container. This motion generates kinetic energy, which is proportional to the speed and mass of the particles. In contrast, solids have the lowest kinetic energy because their particles are tightly packed and have limited movement. The particles in a solid vibrate around a fixed position, and only experience small oscillations. Liquids have an intermediate amount of kinetic energy. The particles in a liquid are less tightly packed than in a solid, and can move more freely, resulting in more kinetic energy. However, liquids have more intermolecular forces between the particles compared to gases, which restricts their movement and reduces their average speed. Therefore, of the three states of matter, gases have the most kinetic energy, followed by liquids and then solids.

Learn more about kinetic energy here:

https://brainly.com/question/26472013

#SPJ4

What ionic compound would bond in a 1 to 2 ratio?

Answers

An ionic compound that would bond in a 1:2 ratio would be one in which the cation has a charge of 2+ and the anion has a charge of 1-. This is because the total charge of the compound must be neutral.

An ionic compound is a type of chemical compound that is formed through the transfer of electrons between atoms. In an ionic compound, one or more positively charged ions (known as cations) are attracted to one or more negatively charged ions (known as anions) to form a stable structure. The cations and anions are held together by electrostatic forces known as ionic bonds.

Ionic compounds typically have high melting and boiling points, are usually crystalline solids at room temperature, and are often soluble in water. They can conduct electricity when dissolved in water or when melted, but not when in a solid state. Ionic compounds have a wide range of applications, including as building blocks for ceramics, in the production of batteries, and as components of various chemical processes. Examples of common ionic compounds include sodium chloride (table salt), calcium carbonate (chalk), and potassium iodide (used in medicine).

To learn more about Ionic compound visit here:

brainly.com/question/3222171

#SPJ4

#1)

A 500 Hz triangular wave with a peak amplitude of 50 V is applied to

the vertical deflecting plates of a CRO. A 1 kHz saw tooth wave with a

peak amplitude of 100 V is applied to the horizontal deflecting plates.

The CRO has a vertical deflection sensitivity of 0. 1 cm/V and a

horizontal deflection sensitivity of 0. 02 cm/V. Assuming that the two

inputs are synchronized, determine the waveform displayed on the

screen?

[2 Marks]

Answers

The CRO (Cathode Ray Oscilloscope) will display a triangular wave that is vertically stretched and horizontally compressed.

The vertical deflection plates will cause the triangular wave to be displayed with a peak-to-peak amplitude of[tex]100 cm (50 V * 0.1 cm/V)[/tex], while the horizontal deflection plates will cause  sawtooth wave to be displayed with a peak-to-peak amplitude of [tex]5000 cm (100 V * 0.02 cm/V).[/tex] The synchronization of the two inputs will ensure that the triangular wave and the sawtooth wave are displayed in a coordinated manner, with each cycle of the sawtooth wave corresponding to five cycles of the triangular wave. The resulting display will show a pattern of diagonal lines that gradually rise and then quickly drop back to the starting position, with each line representing a cycle of the sawtooth wave.

To know more about Cathode Ray Oscilloscope, here

brainly.com/question/30675801

#SPJ4

Reflection, refraction, and the formation of images by mirrors and lenses has been successful described by the A) wave model of light. B) ray model of light. C) particle model of light. D) none of the given answers

Answers

The correct answer is B). Reflection, refraction, and the formation of images by mirrors and lenses has been successful described by the Ray Model of Light.

Reflection, refraction, and the formation of images by mirrors and lenses can be explained using the Ray Model of Light, which states that light travels in straight lines, called rays.

As an electromagnetic wave, light travels in straight lines along narrow beams of light, which are referred to as rays. Despite the fact that reflection or refraction can alter its path, light always moves in a straight line.

When light rays reflect off a surface or pass through a lens, the angle of reflection or refraction can be calculated using geometry and the law of reflection/refraction.

Learn more about "Reflection, refraction, and the formation" at : 'Snell's law' https://brainly.com/question/10112549

#SPJ11

If the resulting trajectory of the charged particle is a circle, what is ⍵, the angular frequency of the circular
motion?
Express ⍵ in terms of g, m, and Bo.

Answers

The angular frequency of circular motion is given by the expression:

ω = [tex]\sqrt{qB/m}[/tex]

If the resulting trajectory of the charged particle is a circle, the angular frequency (ω) of the circular motion can be expressed in terms of g, m, and Bo as follows:

ω = [tex]\sqrt{qB/m}[/tex]

where q is the charge of the particle, B is the magnetic field strength, and m is the mass of the particle.

This formula is known as the cyclotron frequency equation.

The circular motion occurs because the magnetic force (F = qvB) on the charged particle is perpendicular to its velocity (v) and results in a centripetal force that keeps the particle in a circular path with a constant speed.

The angular frequency (ω) represents the rate at which the particle completes a full revolution (2π radians) around the center of the circular path per unit of time.

To know more about cyclotron frequency, refer: https://brainly.com/question/12946099

#SPJ11

How does the star formation in spirals compare to the star formation of elliptical galaxies?

Answers

The spiral galaxies are characterized by the arms winding out from a central nucleus while the elliptical galaxies are characterized by their lack of structure or a central bulge.

Star formation refers to the process by which dense areas within molecular clouds in interstellar space, typically lasting tens of millions of years, form newborn stars. It takes a long time for stars to form, and this process is not well understood.

In comparison to spiral galaxies, elliptical galaxies have low star formation.

Furthermore, elliptical galaxies are made up of stars with a wide range of ages, indicating that the star formation process was rapid and early on in their history.

Spiral galaxies have more gas and dust in their disks than elliptical galaxies, and these are the sites of intense star formation.

The arms are believed to be regions of higher density of stars and interstellar material, as well as more significant gravitational interactions among stars, gas, and dust than in the rest of the disk.

Thus, spiral galaxies are sites of ongoing star formation while elliptical galaxies are mainly populated by old and evolved stars that no longer form. Therefore, spiral galaxies have a higher rate of star formation than elliptical galaxies.

Learn more about Star formation here:

https://brainly.com/question/7467199

#SPJ11

a hydrostatic transmission has pump volumtric efficiency 91 %, a pump mechanical efficiency of 93 %, a motor mechanical efficiency of 95%, and a motor volumetric efficiency of 91%. what is the overall efficiency of the hst (in percent)?

Answers

The hydrostatic transmission's overall efficiency in percent can be calculated using the given information as follows:

Given that:

Volumtric efficiency of the pump = 91%Mechanical efficiency of the pump = 93%Mechanical efficiency of the motor = 95%Volumetric efficiency of the motor = 91%

Formula for calculating overall efficiency of HST is given as:

Overall efficiency of HST = pump volumetric efficiency × pump mechanical efficiency × motor mechanical efficiency × motor volumetric efficiency

Substituting the given values in the above formula, we get:

Overall efficiency of HST = 0.91 × 0.93 × 0.95 × 0.91 = 0.7460585 = 74.61%

Therefore, the overall efficiency of the hydrostatic transmission is 74.61% (rounded to two decimal places).

Learn more about hydrostatic transmission: https://brainly.com/question/16103822

#SPJ11

The diagram below shows a sled moving along a smooth, frictionless track.

Answers

The sections 2 and 4 of the track will the sled experience an unbalanced force which is frictionless surface.

As we see from the diagram the track is a frictionless surface where the sled will move. In section 1 and 3, the sled will experience a balanced force because the gravitational force pulling the sled down is equal to the normal force of the track pushing the sled up. While in section 2 and 4, however, the sled will experience an unbalanced force because the gravitational force pulling the sled down will be greater than the normal force of the track pushing the sled up. This will cause the sled to accelerate and move faster. Hence section 2 and 4 will experience an unbalanced force.

To learn more about frictionless click here https://brainly.com/question/30616235

#SPJ1

find the height of the roller coaster using two different methods. round your answers to one decimal place.

Answers

If there were no friction and air resistance, an ideal coaster could go down and up hills at the same height eternally. If you want to round to one or two decimal places, look at the digit immediately decimal point.

Roller coasters can reach what heights?

Almost always, the start of a roller coaster is a vertical plummet. The automobiles are propelled by a motor to a top of a steep hill, after which gravity takes over completely. Typical vertical fall could be anywhere between 50 and 80 metres high.

How are answers rounded to decimal places?

When round a decimal value, there are a few guidelines to remember. To put it another way, it round previous digit down if the final digit is much less than 5. However, you just round the previous figure up if it is 5 or above. Thus, it round number up if 5, 6, 7, 8, or 9 come after the number we are about to round.

To know more about friction visit:
https://brainly.com/question/13000653

#SPJ1

Help! The image characteristics are ____. (2 points)

A concave mirror is shown with curvature positioned at 8 on a ruler that goes from 0 to 14 centimeters. The object is located at 5, and the focal point is located at 6.5.

a
upright, virtual, and smaller

b
upright, real, and same size

c
inverted, virtual, and smaller

d
inverted, real, and same size

Which is correct?

Answers

I believe the answer is a ! good luck!!

A ball rolls across the floor, slowing down with constant acceleration of magnitude . The ball has positive velocity ???? after rolling a distance x across the floor.
Calculate the ball's initial speed ????0 if ????= 4.51 m/s2, ????=11.17 m/s, and x=2.66 m.

Answers

A ball rolls across the floor, slowing down with a constant acceleration of magnitude a = 4.51 m/s2.

The ball has positive velocity v after rolling a distance x = 2.66 m across the floor.

To calculate the ball's initial speed v0 if

v = 11.17 m/s.

The initial velocity of the ball, v0 =?

The final velocity of the ball, v = 11.17 m/s

The acceleration of magnitude a = 4.51 m/s2

Distance travelled, x = 2.66 m

If an object has initial velocity v0, constant acceleration a, and travelled distance x, then its final velocity is given by:

v2 = v0² + 2ax

Here, the ball's initial velocity is v0, and its final velocity is v.

After substituting the given values, we have:

v2 = v0² + 2ax

=> (11.17)²

= v0² + 2(4.51)(2.66)

=> 124.57

= v0² + 25.39

=> v0² = 124.57 - 25.39

=> v0² = 99.18 => v0 = √99.18

=> v0 = 9.96 m/s

Hence, the initial velocity of the ball is v0 = 9.96 m/s.

To know more about velocity:

https://brainly.com/question/29519833

#SPJ11

assuming a total mass of 80 kg (bicycle plus rider), what must be the cyclist's power output to climb the same hill at the same speed? express your answer using two significant figures.

Answers

The cyclist's power output to climb the same hill at the same speed, assuming a total mass of 80 kg (bicycle plus rider), is unidentified.

Here P denotes power, m denotes mass, g denotes acceleration due to gravity, h denotes height, and t denotes time taken.

Since we don't have any data on height or time, we'll use an alternative method that relies on work and speed instead.

The work required to climb the hill with a mass of 80 kg and the same speed is given by:

W = mgh

where W denotes work.

We'll suppose that the cyclist climbs the hill at a speed of v. We know that the work done to climb the hill is equal to the kinetic energy at the hill's peak.

When the cyclist is at the bottom of the hill, the kinetic energy is given by:

[tex]KE=\frac{1}{2}mv^2[/tex]

When the cyclist reaches the peak, the kinetic energy is zero, and the gravitational potential energy is given by

[tex]mgh=\frac{1}{2}mv^2[/tex]

where h is the height of the hill.

Now, let's rearrange the equation to solve for the power required

[tex]\frac{1}{2}mv^2=mgh\\h=\frac{v^2}{2g}}[/tex]

Substituting [tex]\sqrt{2gh}[/tex] in the equation for power, we have:

[tex]P=W/t\\P=\frac{mgh}{t}\\P=\frac{mg\frac{v^2}{2g}}{t}[/tex]

Here m is the total mass of the bicycle and rider, h is the height of the hill, g is the acceleration due to gravity, and t is the time taken.

Therefore it depends on both speed and height of the rider.

A pendulum has length / and a bob of mass m. Which of the following is true of the linear momentum of the bob as it swings through the lowest point? a. It remains constant because momentum is always conserved. b. It increases in magnitude and changes direction. c. It decreases in magnitude and does not change direction. d. It is converted to angular momentum. e. It is converted to kinetic energy.

Answers

The following is true of the linear momentum of the bob as it swings through the lowest point  that it increases in magnitude and changes direction.

What is linear momentum ?

Momentum (more specifically linear momentum or translational momentum) is defined in Newtonian mechanics as the product of an object's mass and velocity. It is a vector quantity with magnitude and direction. If m is the mass of an object and v is its velocity (also a vector quantity), then p = m * v. The kilogram meter per second (kgm/s) is the unit of momentum measurement in the International System of Units (SI), which is equivalent to the newton-second. According to Newton's second law of motion, the rate of change of a body's momentum is equal to the net force acting on it. Momentum depends on the frame of reference, but it is a conserved quantity in any inertial frame, which means that if a closed system

to know more about linear momentum , visit ;

brainly.com/question/27988315

#SPJ1

two blocks with masses 4m and 7m are on a collision course with the same initial speeds vi. the block with mass 4m is traveling to the left, and the 7m block is traveling to the right. they undergo a head-on elastic collision and each bounces back, retracing its original path. find the final speeds of the particles. (enter your answers in terms of

Answers

The final speeds of the particles expressed in terms of the initial velocity are |v1'| = |v1| = 27/8|vi| and |v2'| = |v2| = 27/14|vi|

The conservation of momentum can be applied. The total momentum of the system before the collision is:

P before = m1v1 + m2v2

where m1 and v1 are the mass and velocity of the 4m block and m2 and v2 are the mass and velocity of the 7m block. Since the two blocks have the same initial speed, the momentum before the collision is:

P before = (4m)(-vi) + (7m)(vi)
P before = 3mvi

After the collision, the two blocks bounce back, so their final velocities are:

v1' = -v1
v2' = -v2

where v1 and v2 are the velocities of the blocks after the collision. Using the conservation of momentum again, the total momentum of the system after the collision is:

Pafter = m1v1' + m2v2'
Pafter = -4mv1 - 7mv2
Pafter = -4m(-v1) - 7m(-v2)
Pafter = 4mv1 + 7mv2

Since the collision is elastic, the total kinetic energy of the system is conserved. Therefore, the kinetic energy before the collision is equal to the kinetic energy after the collision:

Kbefore = Kafter

where Kbefore is the kinetic energy of the system before the collision and Kafter is the kinetic energy of the system after the collision. The kinetic energy can be expressed as:

K = 1/2mv²

Therefore, the total kinetic energy of the system before the collision is:

Kbefore = 1/2(4m)(vi)² + 1/2(7m)(vi)²
Kbefore = 27/2m(vi)²

The total kinetic energy of the system after the collision is:

Kafter = 1/2(4m)(-v1)² + 1/2(7m)(-v2)²
Kafter = 1/2(4m)(v1)² + 1/2(7m)(v2)²

Using the conservation of kinetic energy, Kbefore = Kafter:

27/2m(vi)² = 1/2(4m)(v1)² + 1/2(7m)(v2)²

Simplifying, the final velocities can be expressed in terms of the initial velocity:

v1 = 27/8vi
v2 = 27/14vi

Therefore, the final speeds of the particles are: |v1'| = |v1| = 27/8|vi| and |v2'| = |v2| = 27/14|vi|

More on momentum: https://brainly.com/question/30733354

#SPJ11

suppose two protostars form at the same time, one with a mass of 0.5 msun and the other with a mass of 15 msun . which of the following statements are true? A) The 10MSun protostar will have a smaller change in surface temperature during this phase than the 0.5MSun protostar.
B) The 10MSun protostar will reach the main sequence cooler and fainter than the 0.5MSun protostar.
C) The 10MSun star will end its main-sequence life before the 0.5MSun star even completes its protostar stage.
D) The 10MSun protostar will have a smaller change in luminosity during the sequence shown than the 0.5MSun protostar.
E) The 10MSun protostar will be much more luminous than the 0.5MSun protostar.

Answers

The statements that are true are:

B) The 10MSun protostar will reach the main sequence cooler and fainter than the 0.5MSun protostar.

E) The 10MSun protostar will be much more luminous than the 0.5MSun protostar.

To understand why, we need to know that a star's mass determines its temperature, luminosity, and lifespan. More massive stars are hotter, more luminous, and have shorter lifespans than less massive stars.

In this case, the 15MSun protostar is much more massive than the 0.5MSun protostar. Therefore, statement B is true because the 10MSun protostar is closer in mass to the 15MSun protostar, and therefore it will have a higher temperature during the protostar phase, making it cooler and fainter when it reaches the main sequence.

Statement E is true because more massive stars are more luminous than less massive stars. The 10MSun protostar is closer in mass to the 15MSun protostar, and therefore it will be much more luminous than the 0.5MSun protostar.

Hence b and e are correct option.

For more similar questions on stars and astrophysics:

brainly.com/question/27996005

#SPJ11

if two tiny identical spheres attract each other with a force of 2.0 nn when they are 29 cm apart, what is the mass of each sphere? express your answer with the appropriate units.

Answers

The mass of each sphere with the appropriate units are the  0.6 kg by the two tiny identical spheres attract each other with a force of 2.0 nn when they are 29 cm apart.

Let's consider the following scenario: Two tiny identical spheres attract each other with a force of 2.0 nn when they are 29 cm apart. The mass of each sphere is what we need to calculate. The formula for calculating the mass of each sphere. F = Gm1m2 / r²Where:F = Force. G = Gravitational constantm1 and m2 = the masses of the object sr = the distance between the objects.

Substitute the given values: Force (F) = 2.0 nn. Distance (r) = 29 cm = 0.29 m. Gravitational constant (G) = 6.67 × 10-11 N.m²/kg²Find the mass of each sphere.m1 = m2 = m. Multiply the entire equation by ][tex]r² / G:m² = F × r² / G = (2.0 nn) × (0.29 m)² / 6.67 × 10-11 N.m²/kg²= 0.6 kg.[/tex]

Therefore, each sphere's mass is 0.6 kg.

Read more about mass:

https://brainly.com/question/19385703

#SPJ11

calculate the fluid force on one side of a 5ft by 5ft square plate if the plate is at the bottom of a pool filled with water to a depth of 8ft and, a. lying flat on its 5ft by 5ft face. b. resting vertically on a 5 ft edge c. resting on a 5 ft edge and tilted at 45 degrees to the bottom of the pool.

Answers

The fluid force on one side of the 5ft by 5ft plate lying flat on its 5ft by 5ft face, we can use the equation F = ρghA and the F = 1,945,000 N.

What is the fluid force?

Since the plate is at the bottom of the pool and the pool is filled with water to a depth of 8ft, h = 8ft and the area of the plate A = 5ft × 5ft = 25ft².

There F is the fluid force, ρ is the density of the fluid, g is the acceleration due to gravity, h is the depth of the fluid, and A is the area of the plate.

Therefore, the fluid force on one side of the plate is:

F = ρghA = 1000kg/m³ × 9.81m/s² × 8ft × 25ft² = 1,945,000 N.

To calculate the fluid force on one side of the plate resting vertically on a 5 ft edge, we can use the same equation as above but with an area equal to the length of the plate multiplied by the depth of the pool (8ft):

F = ρghA = 1000kg/m³ × 9.81m/s² × 8ft × 5ft = 392,000 N.

To calculate the fluid force on one side of the plate resting on a 5ft edge and tilted at 45 degrees to the bottom of the pool, we can use the same equation as above but with an area equal to the length of the plate multiplied by the depth of the pool multiplied by the cosine of the angle of the plate:

F = ρghA = 1000kg/m³ × 9.81m/s² × 8ft × 5ft × cos(45) = 283,200 N.

Read more about the force here:

https://brainly.com/question/12785175

#SPJ11

Which of the following statements describes a method by which signaling via TGF-beta and BMP receptors is decreased ? I SMAD-mediated expression of a protein that acts in the nucleus to block SMAD-dependent signaling II SMAD-mediated expression of a SMAD that cannot be phosphorylated by a TGF-beta/BMP receptor complex. III SMAD-mediated histone acetylation of genes whose expression are regulated by SMADs. IV SMAD-dependent phosphorylation of the cytoplasmic receptor tails of type-I TGF-beta/BMP receptors. O I, II and III O I, II, III and IV III and IV I only Olandi O I and II

Answers

The following statements describes a method by which signaling via TGF-beta and BMP receptors is decreased is SMAD-mediated expression of a protein that acts in the nucleus to block SMAD-dependent signaling. The correct option is I.

TGF-beta or transforming growth factor-beta is a peptide growth factor that belongs to the cytokine superfamily, it is a potent regulator of cellular proliferation, differentiation, and other physiological activities. It is found in many cells, including bone cells, and participates in the formation and maintenance of bone tissue, in addition to its role in immune function.

Bone morphogenetic proteins (BMPs) are part of the transforming growth factor-beta superfamily of proteins. BMPs are essential for embryonic development, and their primary function is to regulate bone formation. SMADs are proteins that transmit signals from the cell surface to the nucleus in response to a variety of extracellular signals, including transforming growth factor-beta (TGF-beta) and bone morphogenetic protein (BMP). The SMAD family of transcription factors is composed of several members, including receptor-regulated SMADs, common-partner SMADs, and inhibitory SMADs.

Learn more about cytokine at:

https://brainly.com/question/12199920

#SPJ11

An object is_____ if its position changes relative to another object.
A. in motion
B. at reset
C. a frame of refence
D. magical

Answers

An object is "in motion" if its position changes relative to another object.

Motion is a fundamental concept in physics, which describes how objects move and change position over time. When we say that an object is in motion, we mean that it is changing its position with respect to some reference point or frame of reference.

A reference point is a fixed point in space that we use as a point of comparison to measure an object's position and motion. For example, when we say that a car is moving on a highway, we are using the highway as a frame of reference to measure the car's motion.

An item is considered to be "in motion" if its position in relation to another object changes.

To learn more about relative position refer to:

brainly.com/question/29153175

#SPJ4

if the total mass is m , find the moment of inertia about an axis through the center and perpendicular to the plane of the square. use the parallel-axis theorem. express your answer in terms of the variables m and a .

Answers

The moment of inertia of the square about an axis through the center and perpendicular to the plane of the square is I = m a²/3.

Step by step explnation:

The moment of inertia about an axis through the center and perpendicular to the plane of the square can be found using the parallel-axis theorem. The moment of inertia about the center of the square is [tex]I_c_m[/tex] = (m a²)/6.

Using the parallel-axis theorem, the moment of inertia about an axis through the center and perpendicular to the plane of the square is I = [tex]I_c_m[/tex] + m a² = ma²/3.

Thus, the moment of inertia of the square about an axis through the center and perpendicular to the plane of the square is I = m a²/3.

Learn more about moment of inertia at : https://brainly.com/question/14119750

#SPJ11

The bottom wave has a greater _____

(1 point)


amplitude.


frequency.


trough.

wavelength.

Answers

Answer:

frequency

because more wavelength is covered in 1 second in the bottom one

Module 25: AC Circuits Learn Question An inductor with inductance L is connected in series with an AC source that provides a sinusoidal voltage of v of t is equal to V times cosine of begin quantity omega times t end quantity, where V is the maximum voltage, omega is the angular frequency, and t is the time. If a second identical inductor is wired in series with the first inductor, what happens to the total inductive reactance, XL, of the circuit?
XL decreases by a factor of 2.
XL increases by a factor of 4.
XL decreases by a factor of 4.
XL increases by a factor of 2.

Answers

The total inductive reactance, XL, of the circuit increases by a factor of 2 when a second identical inductor is wired in series with the first inductor. Thus, option d is correct.

Inductive reactance is the opposition of an inductor to a change in current that produces a magnetic field. Inductive reactance is the inductive equivalent of resistive impedance.

It is measured in ohms and can be calculated using the following formula:

XL = 2πfL

where XL is the inductive reactance, f is the frequency of the applied voltage, and L is the inductance of the coil.

Since the two inductors are identical and the frequency of the AC source does not change, the total inductance of the circuit doubles, resulting in a factor of 2 increase in inductive reactance (XL). Thus, option d is correct.

Learn more about reactance here:

https://brainly.com/question/988958

#SPJ11

Light of 630 nm wavelength illuminates two slits that are 0.25 mm apart. FIGURE EX33.5 shows the intensity pattern seen on a screen behind the slits. What is the distance to the screen?

Answers

The distance to the screen from the two slits is 4.0 meters

Distance is the total distance traveled by an object over a specific time interval. 

The distance can be calculated using the equation d = λ/(2a), where

d is the distance to the screen, λ is the wavelength of the light (630 nm in this case), and a is the separation of the two slits (0.25 mm in this case).

Plugging these values in, we get: d = 630 nm / (2 * 0.25 mm) = 4.0 m. The distance to the screen from the two slits is 4.0 meters, as seen in Figure EX33.5.

Learn more about term Distance: brainly.com/question/26550516

#SPJ11

gasoline has an energy density of roughly 122.8 megajoules per gallon. a car company has produced a new economy car that has a fuel economy of 35 miles per gallon, and 0.877 mj is exerted per mile to power the forward motion of the vehicle. what is the percent fuel efficiency of the car?

Answers

The percent fuel efficiency of the car which has gasoline is about 14.8%.

What is Percent fuel efficiency?


Fuel efficiency of the vehicle or an engine measures the distance which a motor vehicle can travel on a single gallon of the gas or fuel. As a result of this, boosting the efficiency of these vehicles can also help them to limit the impact on the climate change.

Energy density of gasoline = 122.8 mega joules per gallon

Fuel economy of the car = 35 miles per gallon

Energy exerted per mile = 0.877 mega joules

Percent fuel efficiency = Energy output of car/ Energy input of car × 100

Where, Energy input of car = Energy density of gasoline × Number of gallons of gasoline used

Energy input of car = 122.8 mega joules/gallon × 1 gallon/ 35 miles × 1 mile/ 1.60934 kilometers× 1.60934 kilojoules/ mega joule

Energy input of car = 4.45 mega joules per kilometer

Energy output of car = Energy exerted per mile × Number of miles traveled

Energy output of car = 0.877 mega joules per mile × 35 miles

Energy output of car = 30.8 mega joules

Percent fuel efficiency = 30.8 mega joules/4.45 mega joules × 100

Percent fuel efficiency = 14.8%

Therefore, percent fuel efficiency of the car is 14.8%.

Learn more about Percent fuel efficiency here:

https://brainly.com/question/14366094

#SPJ11

Other Questions
can anyone help me with this question triangles? A wire first bent into the shape of a rectangle with width 5cm and lenth 11 cm.then the wire is unbent and reshaped into a square what is the length kf a side of the square if you witnessed a crime and were asked to pick out the person who committed the crime from a lineup of six people, the other five people are called Please help...Experimental Probability and Theoretical Probability..I'm getting the wrong answer A baseball team has home games on Thursday and Sunday. The two games together earn $4064.50 for the team. Thursday's game generates $400.50 less than Sunday's game. How much moneywas taken in at each game? The fibrous protein that winds alone the groove of the F actin double helix and blocks the myosin-binding sites on the actin filaments is called _______________. based on the information in the lesson on prayer and fasting, which of the following are reasons for praying? luoa Culinary Arts questions 31. What does cross-contamination mean in a kitchen environment? Imagine that you are in a kitchen environment where you have been asked to filet some fish and then switch over to dressing salads that are ready for service. How would you prevent cross-contamination between these foods?2. What is a food-borne illness that you have heard about recently that might have resulted in the recall of food products or a restaurant being briefly closed? Go online to learn more about that food-borne illness, how it was handled, and how it could have been prevented. 3. Describe your own personal hygiene habits and analyze how they compare to the expected personal hygiene standards for kitchen workers. Identify the main components of personal hygiene procedures that kitchen workers must adhere to in order to comply with sanitation laws and standards, including how to properly wash hands. 4. Imagine that one of your kitchen workers tells you that he believes your kitchen has a pest problem and might be facing an infestation. While you trust this employee, you want to see for yourself. What would you look for in trying to determine if you have a pest problem or not? Describe the key signs that you would look for and what you would do if you determined that you were suffering from a pest problem. 5. If you were asked to create a new sanitation plan for the kitchen that you work in, what would you understand this task to entail? What are some of the common safety and sanitation standards that would be crucial to include in your plan? What are some standards for infectious disease control and what role would these play in your plan? Explain Different patterns of inheritance Inheritance of traits is often more complex than just simple dominant recessive relationships between alleles. Non-Mendelian inheritance is any pattern of inheritance in which traits do not segregate in accordance with Mendel's laws. Below is a list of examples. Please correctly classify each of the following traits according to the type of Non-Mendelian inheritance pattern. Multiple genes influence a phenotype In A and B blood types 5 This means that when an organism has two different alleles (ie, is a heterozygote). it'll express both at the same time, When two different alleles are inherited, both traits are expressed at the same time but the traits produce an intermediate phenotype rather than a dominant trait masking a recessive trait Traits are controlled by many genes instead of traits controlled by alleles from one gene Two alleles are both expressed equally rather than a dominant allele taking complete control over a recessive allele. Codominance Height Two genes can interact to produce a phenotype, such that one gene can override another Human eye color Familial hypercholesterolemia nce pattern. Multiple genes influence a phenotype Incomplete dominance Codominance Epistatic interaction Reset The supervisor at vector control wants your report of how the mosquito population is growing. Do you think your report is better supported by the table above or by a graph of the growth function as show below? WHY? Explain the reasoning behind your conclusion. Rick the receptionist claims that he types an average of 70 words per minute. In order to test this, Rick's boss gives him 15 different one minute typing challenges throughout the day. On each challenge, Rick's boss records the number of words typed. The boss calculates the sample mean to be 67 words per minute and the sample standard deviation to be 16 words per minute. He calculates a test statistic for the following hypothesis test:H0: ?\u = 70Ha: u (does not equal) 70Assuming the null hypothesis is true, this test statistic will follow:a. the t distribution with 16 degrees of freedomb. the t distribution with 14 degrees of freedomc. the t distribution with 15 degrees of freedomd. the standard normal distribution(the answer I have chosene. the t distribution with 14 degrees of freedom which molecules are bound to hemoglobin when hemoglobin is in the r state?a. Fe3+ b. CO2 c.O2 d. 2,3-bisphosphoglycerate Assuming that all is working properly, which of the following is at a higher energy level? 10 points! HELP ASAP PLEASE HELP ME FIND THE AREA and the perimeter Find the value of x to the nearest tenth. which statement about apprenticeships is true?multiple choicea. an apprenticeship does not include any classroom training methods.b. an apprenticeship cannot be sponsored by groups outside the firm.c. an apprenticeship is mostly used for teaching management and interpersonal skills.d. an apprentice can earn an income while learning a trade.e. an apprenticeship does not offer hands-on learning and practice. Without doing a calculation, predict which of these compounds has the greatest molar solubility in water . A) AgBr (Ksp = 5.0x10-13)B) Agl (Ksp = 8.3x10-17)C) AgCl (Ksp = 1.8x10-10) problem 8-68 (lo 8-3) (algo) skip to question [the following information applies to the questions displayed below.] eva received $62,000 in compensation payments from jazz corporation during 2022. eva incurred $5,500 in business expenses relating to her work for jazz corporation jazz did not reimburse eva for any of these expenses. eva is single and deducts a standard deduction of $12,950. based on these facts, answer the following questions: use tax rate schedule for reference. note: leave no answer blank. enter zero if applicable. round your intermediate and final answers to the nearest whole dollar amount. I tried it for about half an hour and I don't know I just don't get it The energy you need and receive from food is measured in nutrients. True or false